A quick chess

Logic Level 5

Determine the minimum number of half-moves required to reach the given position.

Notes:

  • One half-move (also known as ply) is a movement done by a single player (either White or Black). One full move, a White move followed by a Black move, is composed of two half-moves.
  • Both sides cooperate to reach the given position in the fewest number of half-moves possible.


The answer is 15.

This section requires Javascript.
You are seeing this because something didn't load right. We suggest you, (a) try refreshing the page, (b) enabling javascript if it is disabled on your browser and, finally, (c) loading the non-javascript version of this page . We're sorry about the hassle.

2 solutions

Richard Polak
Jun 5, 2015

Proof that the number of half-moves, say, n, is greater or equal than 15.

We will show that the black queen needs at least 7 half-moves, and that then at least one more white half move is needed.

  1. d6 and Qd7 are obviously necessary. - 2 half-moves.

  2. The queen has to capture a knight, two bishops and the other queen. - 4 half-moves.

  3. We state that, if the queen captures at every half-move (after the second) the fourth piece to be captured has to be the queen's bishop (so that she has to go to e1 in order to be captured by the king, adding the last half-move).

Assume this statement is not true. Then, the bishop has to be captured at the third move (it is easy to see that it is impossible to get to c1 in less than 3 moves). There is only one way to get to c1 in three moves from Qd7: Qg4 Qd1 Qc1, implying that there were pieces to be captured in those squares. This leads to only three possible situations:

1. e3 e6
2. Qg4 Qd7
3. Be2 Qxg4
4. Bd1 Qxd1+

and the King has to capture the Queen.

1. e3 e6
2. Be2 Qd7
3. Bg4 Qxg4
4. Nh3 Qxd1+

and the King has to capture the Queen, or

...4. Kf5 Qxd1++

obviously not the case.

Hence, as we stated, the black needs to do at least 7 half-moves, plus the last half-move by the white King, 7 2 + 1 = 15 n 15 7\cdot2+1=15\implies\boxed{n\geq15}

Note: I assumed it is the King who captures the Queen, since there is no way that the Queen ends in h1 or b1. I also assumed black doesn't use the knights.

We will show that the black queen needs at least 7 half-moves

What makes you sure that it's the queen that performs the captures, not some bishop/knight that made captures and returned to the original square?

Ivan Koswara - 6 years ago

Log in to reply

The (free) white bishop obviously can't capture on c1. The knights clearly have no time to come back to their starting positions, I actually say it in the solution ;)

Richard Polak - 5 years, 11 months ago
Ivan Koswara
Jun 5, 2015

The fastest game leading to it has 15 half-moves; moreover, it is unique:

  1. e3 d6
  2. Bb5 Qd7
  3. Ne2 Qxb5
  4. 0-0 Qxe2
  5. f3 Qxd1
  6. Kf2 Qxc1
  7. Rh1 Qe1+
  8. Kxe1

This also has a theme: White castled, but then returned the king and rook back, for an uncastling theme .

This kind of puzzles, where you have to construct a game leading to a given position, is called a proof game . (Normally, proof games give you the number of half-moves you have, and you need to construct the game leading to it; for obvious reasons it's difficult to duplicate here, so you get "find the minimum number of moves" instead.) This composition is made by Andrew Buchanan, 2005.

Moderator note:

Interesting problem.

How can one show that the minimum number of moves is indeed 15? Did it involve a computer search of various possible moves?

Beautiful problem , and well said @Ivan Koswara ,chess is really "more than just a game."

All chess players Here

Siddharth Bhatnagar - 6 years ago

Wait... why is it a good idea for White to NOT capture BLACK Queen in step 3?

Pi Han Goh - 6 years ago

Log in to reply

Because the objective is not for either side to win; the objective is to reach the position together as fast as possible. Chess is more than just a game.

Ivan Koswara - 6 years ago

@Challenge Master: Yes, I used a computer search to do this. The original problem states that this is a proof game in 7.5 moves (= 15 half-moves), and clearly we need at least 3 half-moves (White's e- and f-file pawns have moved, requiring at least two moves from White), so I just tested with a special program that the given position is not achievable in n n half-moves for each n = 3 , 4 , , 14 n = 3, 4, \ldots, 14 .

Ivan Koswara - 6 years ago

0 pending reports

×

Problem Loading...

Note Loading...

Set Loading...